round your answer to the nearest hundredth. Find angle A=?

Round Your Answer To The Nearest Hundredth. Find Angle A=?

Answers

Answer 1

Answer:

A=48.81

Step-by-step explanation:

it is a right angle triangle find the hypotenuse c using Pythagorean theorem:

c²=a²+b²

c²=8²+7²

c=√64+49

c=10.63

sin A =opp/hyp

sin A=8/10.63

A=  48.81

another way :

tan A=opp/adj

tan A=8/7

A=48.81


Related Questions

Need help with mark brainlist.

Nam worked on a job for 10 days. On each of the last 2 days, he worked 2 hours more than the mean number of hours he worked per day during the first 8 days. If he worked 69 hours in all, how many hours did he work during the last 2 days together?

Answers

Answer: 17 hours

Step-by-step explanation:

Given that On each of the last 2 days, he worked 2 hours more than the mean number of hours he worked per day during the first 8 days. That is he worked additional 4 hours for the two days.

Let the total hours for the 8 days = E

The mean = E/8 = 0.125E

For the two last days, he worked

( 0.125E + 2 ) × 2 = 0.25E + 4

If he worked 69 hours in all, then

E + 0.25E + 4 = 69

Collect the like terms

1.25E = 69 - 4

1.25E = 65

E = 65/1.25

E = 52.

Now find the mean of the first 8 days

Mean = 52 / 8 = 6.5 hours

Nam works during the last 2 days together for:

(6.5 + 2)×2

8.5 × 2 = 17 hours

What is the simple interest at the end of two year for a loan of rupees 20000 at 10% intrest annum

Answers

Answer:

4000 rupees

Step-by-step explanation:

20000×10×2/100

= 4000 rupees

The National Weather Service collects data on the number of hours of consecutive rainfall and the number of minor traffic accidents in a particular city. The scatter plot shows the data it gathered and the line of best fit. For a school project, Peyton uses technology to calculate the equation for line of best fit. If Peyton's calculation is correct, which equation could represent the line of best fit for this data?

Answers

The equation that could best represent the line of best fit for the given data is; y = 0.625x

How to find a linear equation from a scatter plot?

The formula for the equation of a line in slope intercept form is;

y = mx + c

where;

m is slope

c is y-intercept which is the point where the line intersects the y-axis

Now, in this question, we see that the line intersects the y-axis at 0. Thus;

y-intercept; c = 0

Now, let us find the slope from the formula;

m = (y₂ - y₁)/(x₂ - x₁)

Using the first and penultimate coordinate which are;

(0, 0) and (8, 5), we have;

m = (5 - 0)/(8 - 0)

m = 0.625

Thus, the equation is;

y = 0.625x

Read more about Scatter plot equation at; https://brainly.com/question/6592115

#SPJ1

Answer:

y=0.625x

Step-by-step explanation:

plato

rearrange the equation so the r is the independent variable

Answers

Answer:

q = 3+1/2 r

Step-by-step explanation:

10q -5r = 30

Add 5r to each side

10q -5r+5r = 30+5r

10q = 30 +5r

Divide by 10

10q/10 = 30/10 +5r/10

q = 3+1/2 r

2. Find the missing angle x.
155°
60°
X

Answers

Answer:

exterior angle property use ...

the exterior angle of an angle is equal to sum of rest 2 interior angles in a triangle

155 = 60 + x

x = 155 - 60 = 95

The missing angle x in the triangle is 95 degrees.

Here, we have,

from the given figure we get,

one angle of the triangle is: 180- 155 = 25 degrees.

To find the missing angle x in the triangle, we can use the fact that the sum of the angles in a triangle is always 180 degrees.

In this case, we are given two angles of the triangle: 25 degrees and 60 degrees.

Let's denote the missing angle as x.

To find x, we can subtract the sum of the given angles from 180 degrees:

x = 180° - (25° + 60°).

Simplifying the expression inside the parentheses:

x = 180° - 85°.

Subtracting:

x = 95°.

Therefore, the missing angle x in the triangle is 95 degrees.

To learn more on triangle click:

brainly.com/question/29126067

#SPJ4

Tell me the answer
69:?::89:36​

Answers

?????????????????????????????????????

Calculate : y
y = 20
y = 10
y = 15
y = 144​
choose the only correct answer

Answers

Answer : y = 20 ( Use Pytago’ Theorem )

Solve: 3a^2-4b a= -6 b= -5 If you could also leave an explanation that would be great! Thank you for your time!

Answers

Answer:

128

Step-by-step explanation:

3a² - 4b

plug in values

3(-6)² - 4(-5)

use PEMDAS and simplify (-6)² first

3(36) -4(-5)

multiply

108 + 20

add

128

hope this helps :)

Suppose y varies jointly as x & z. If y = -180 when z = 15and x = -3,then find y when x = 7 and z = -5. 1 point

Answers

Answer:

y = - 140

Step-by-step explanation:

The statement

y varies jointly as x & z is written as

y = kxz

to find y when x = 7 and z = -5 we must first find the relationship between the variables

when y = - 180

z = 15

x = - 3

- 180 = k(15)(-3)

-180 = - 45k

Divide both sides by - 45

k = 4

The formula for the variation is

y = 4xz

when

x = 7

z = -5

y = 4(7)(-5)

y = 4(-35)

y = - 140

Hope this helps you

Donavan and Jones are randomly choosing chalices to drink from. 4 of the chalices contain purified water, 5 contain spoiled milk, and 6 contain diluted flat soda. Jones gets to choose a chalice from the remaining ones after Donavan drinks from 3 of them. If the chances of Jones choosing a chalice with purified water is then 1/3, how many of the chalices that Donavan drank out of contained purified water?

Answers

Answer:

0

Step-by-step explanation:

Chalice with purified water = 4

With spoiled milk = 5

With flat soda = 6

Probability of Jones drinking from a chalice with purified water after Donovan had drank from 3 = 1/3

Probability = required outcome / Total possible outcomes

Total possible outcomes = (4 + 5 + 6) = 15

After Donovan drinks 3 :

Total possible outcomes = (15 - 3) = 12

If the probability of Jones choosing a chalice with purified water is 1/3 then :

(Required outcome / Total possible outcomes) =1 /3

(Required outcome / 12) = 1 / 3

Required outcome * 3 = 12

Required outcome = 12 / 3

Required outcome = 4

Therefore, since initial number of chalice with purified water is 4,

4 - 4 = 0

Then Donovan did not drink from a chalice containing purified water.

I want you to help me with this

Answers

Answer:

$44.1

Step-by-step explanation:

March>>>>$40

for April: $40 × 5/100= $2 which is the 5% of $40.

Meaning for April Siti got $42

Then for May: $42 × 5 /100= $2.1 meaning she got $44.1 for May.

-3 find the value. -3 ( 2 ) ² x (4)

pls tell the answer step by step pls ​

Answers

(-2/3)^(-2) × x × (3/4)-3

(-3/2)^2 × x ×(4/3)^3

-3/2×-3/2×4/3×4/3×4/3×x

= 16/3 × x

= 16x/3

Must click thanks and mark brainliest

A quadratic polynomial whose zeroes are 5 and -3 is

Answers

Answer:

Well, we can write a really simple quadratic polynomial as:

P(x) = a*(x - c1)*(x - c2)

Where a is a constat, and c1 and c2 are the roots, here we have:

c1 = 5 and c2 = -3

P(x) = a*(x -5)(x + 3) = a*x^2 + a*x*3 - a*5*x - a*15

then the quadratic polynomial can be:

P(x) = a*x^2 + a*(3 - 5)*x - a*15

P(x) = a*x^2 - 2*a*x - a*15

Where a can be any real number.

Resolve 36x2 – 81y2 into factors

Answers

Answer:

(6x + 9y)(6x - 9y)

Step-by-step explanation:

We can use the difference of squares which states that a² - b² = (a + b)(a - b), in this case, a = 6x and b = 9y so the answer is (6x + 9y)(6x - 9y).

The height h (in feet) of an object t seconds after it is dropped can be modeled by the quadratic equationh = -16t2 + h0, where h0 is the initial height of the object. Suppose a small rock dislodges from a ledge that is 255 ft above a canyon floor. Solve the equation h = -16t2 + 255 for t, using the quadratic formula to determine the time it takes the rock to reach the canyon floor.

Answers

Answer:

The time it takes the rock to reach the canyon floor is approximately 4 seconds.

Step-by-step explanation:

The equation representing the height h (in feet) of an object t seconds after it is dropped is:

[tex]h=-16t^{2}+h_{0}[/tex]

Here, h₀ is the initial height of the object.

It is provided that a small rock dislodges from a ledge that is 255 ft above a canyon floor.

That is, h₀ = 255 ft.

So, when the rock to reaches the canyon floor the final height will be, h = 0.

Compute the time it takes the rock to reach the canyon floor as follows:

[tex]h=-16t^{2}+h_{0}[/tex]

[tex]0=-16t^{2}+255\\\\16t^{2}=255\\\\t^{2}=\frac{255}{16}\\\\t^{2}=15.9375\\\\t=\sqrt{15.9375}\\\\t=3.99218\\\\t\approx 4[/tex]

Thus, the time it takes the rock to reach the canyon floor is approximately 4 seconds.

Answer:

t=4

Step-by-step explanation:

ed2020

pls help asap .........................

Answers

Answer:

40

Step-by-step explanation:

We can split this figure into two figures, a square and a trapezoid. The square, as shown has a side length of 4. That means that the area of the square is 16. The trapezoid, has base lengths of 2 and 4, and a height of 8. We can plug that into the area of a trapezoid formula:

[tex]\frac{2+4}{2}(8)[/tex]

Two plus 4 is 6, and 6 divided by 2 is 3. Three times 8 is 24.

24 plus 16 (the area of the square) is equal to 40, which is the area of the shape.

The circumference of a circle is 6π in. What is the area, in square inches? Express your answer in terms of \piπ.

Answers

Step-by-step explanation:

Hey, there!!

According to the question,

There is given circumference = 6pi in.

or, 2.pi.r = 6pi.in

or, r = 6 pi.in/ 2

Therefore, the radius (r)= 3in.

now,

area = pi.r^2

a= pi× (3in)^2

Therefore, the area is 9.pi.in^2.

Hope it helps....

Answer:

A ≈ 2.86

Step-by-step explanation:

Using the formulas

A = πr²

C = 2πr

Solving for A

A = C²/ 4π = 6²/ 4·π ≈ 2.86479

Given an angle of a triangle and the opposite side length; which trigonometric function would you use to find the hypotenuse? a TAN b COS c SIN d Not enough information

Answers

Answer:

Sin

Step-by-step explanation:

Sin < = opposite/hypotenuse


Find the surface area of this cylinder.
Round to the nearest tenth.
p= 5 cm
5 cm
SA = [ ? ] cm?

Answers

Answer:

314.2

Step-by-step explanation:

2πr²+2πrh

= 2×π×5²+2×π×5×5

= 100π

= 314.2 cm²

Answer:

Does the answer help you?

Two weeks in a row, the golf course hosts a group of golfers. The second week had 10 more golfers than the first week. Use the dot plots to choose the true statement.

Answers

Answer:

The second week had 10 more golfers than the first week.

Step-by-step explanation:

The graph clearly shows that the second week golfer are more than the first week. The median for week 2 is greater than the median of week 1. The graph of week 2 shows that number of golfers had increased and their age is between 60 to 80 years of age. The median of week 2 is 79 where as median of week 1 is 75.

Sin³xcosx+sinxcos³x=sinxcosx

Answers

Answer:

Step-by-step explanation:

[tex]sin^3xcosx+sinxcos^3x\\=sinx cosx(sin^2x+cos^2x)\\=sinx ~cosx(1)\\=sin~x~cos~x[/tex]

determine the image of the point p[-3,10) under the translation [5,-7]

Answers

[tex](-3+5,10-7)=(2,3)[/tex]

Write an expression that can be used to find the price of a television that is on sale for 20% off the regular price of p dollars. Can you write a second expression equivalent to the one you wrote in the last questions.

Answers

Answer:

The expression that could help calculate the price of the TV is;

$P - 20% of $P

Step-by-step explanation:

Here, we want to write an expression that corresponds to the price of a television set that is on sale at a price which is 20% off the regular price.

From the question, we can see that the regular price is $P

So now we are having 20% off;

This corresponds to;

20/100 * p = p/5 = 0.2p

So in the expression form, we can have;

$P - 20% of $P

Las dos compañías cobran el mismo precio por viaje, cada viaje vale $100.00 pesos, los pobladores dicen que uno de los camiones tiene una capacidad de 3 toneladas y el otro de 4 toneladas, en total se hicieron 23 viajes y se transportaron 80 toneladas de tierra.

Answers

Huh I'm confused I'll try & translate

a number when divided by 10 leaves the remainder 5. If the same number is doubled, and divided by 10, the new remainder is _____

Answers

Answer:

10

Step-by-step explanation

50 divided by 10 is 5 so

50 times 2 which is 100 divided by 10 is 10

Part C Next, find the length of BC place point F at (4,4) and draw BF and FC now you hav the right ∆BFC with BC as the hypotenuse find BC and FC ising the coordinates of B,C, and,F then use the Pythagorean theorem to find BC show your work need help ASAP giving thanks and points away I just need these answer fast ???

Answers

Answer:

BF = |4 – 1| = 3

FC = |4 – 0| = 4

Using the Pythagorean Theorem to find BC:

BC2  =  BF2 + FC2

BC2   =  32 + 42

BC2   =  9 + 16

BC  =  sqaure root 25

BC  =  5

Step-by-step explanation:

The length of BC is 5 unit.

What is Pythagoras Theorem?

If ABC is a triangle with AC as the hypotenuse and angle B with 90 degrees then we have:

|AC|^2 = |AB|^2 + |BC|^2    

where |AB| = length of line segment AB. (AB and BC are rest of the two sides of that triangle ABC, AC being the hypotenuse).

We need to find the length of BC place point F at (4,4) and draw BF and FC now you have the right ∆BFC with BC as the hypotenuse find BC and FC using the coordinates of B,C, and,F then use the Pythagorean theorem to find BC.

Using the Pythagorean Theorem to find BC:

BC^2  =  BF^2 + FC^2

BC^2   =  32 + 42

BC^2   =  9 + 16

BC  =  √25

BC  =  5

So, BF = |4 – 1| = 3

FC = |4 – 0| = 4

Learn more about Pythagoras theorem here:

https://brainly.com/question/12105522

#SPJ2

NEED HELP ASAP!!!!!!!

Answers

Letter b because you need to multiply

Solve for x. Round your answer to the nearest tenth if necessary.

X=

Answers

Answer:

7.2

Step-by-step explanation:

Since the angle is equal, 24/(x+8.7)=13.1/8.7. x=7.2

Answer:

x ≈ 7.2

Step-by-step explanation:

Δ MNO and Δ MKL are similar, so the corresponding sides are in proportion, that is

[tex]\frac{MN}{MK}[/tex] = [tex]\frac{MO}{ML}[/tex] , substitute values

[tex]\frac{13.1}{13.1+10.9}[/tex] = [tex]\frac{8.7}{8.7 +x}[/tex]

[tex]\frac{13.1}{24}[/tex] = [tex]\frac{8.7}{8.7+x}[/tex] ( cross- multiply )

13.1(8.7 + x) = 208.8 ← distribute left side

113.97 + 13.1x = 208.8 ( subtract 113.97 from both sides )

13.1x = 94.83 ( divide both sides by 13.1 )

x = [tex]\frac{94.83}{13.1}[/tex] ≈ 7.2 ( to the nearest tenth )

Challenge: For a particular job, the Amax Employment Agency charges a fee that is equal to 15% of the first
month's pay. If the job pays X dollars annually, express the agency fee algebraically.

Answers

Answer:

0.0125x or x/80

Step-by-step explanation:

Salary: x dollars per year

To find the pay per month, we divide the annual pay by 12.

The monthly pay is x/12

15% of the first month's salary is

15% of x/12 = 0.15 * x/12 = 0.0125x = x/80

Answer: 0.0125x

Find the simple interest on ₹ 4000 at 7.5 p.a. for 3 years 3 months. Also find the amount.

Answers

Step-by-step explanation:

it is simple you know

make 3 month into year by 3/12

then solve it using formula

Other Questions
Newscaster Nick, age 60, is fired from his job anchoring the local news. He argues that it is b/c of his age, and that the station manager wanted to go with the younger ancho. The manager says that he replaced Nick b/c the other anchor draws substantially more viewers, which increases the station's revenues & that Nick's age had nothing to do with it.Nick sues under the ADEA. He will win if...a.) He can show that his age was the only reason for being firedb.) he can show that his age was one significant reason for being fired.c.) He draws just as many viewers as his replacement.d.) None Here, Equiano is sold againTo a woman and a boy his age What valuedo you suppose slave ownersplaced on slaves' lives?(Grade 9-10 Review) Score for Question 3: of 5 points). What is the area of triangle BCD to the nearest tenth of a square centimeter? Use special right triangles tohelp find the height. Show your work. Plot can be broken into several stages:action, and conclusion.__, rising action, climax, fallingO A. anticlimaxB. contrivanceC. expositionD. story Are my answers correct for this?My answers:1) I *HAD* a really terrible job interview.2) Once, I *FORGOT* to switch off my phone when I was at the Cinema.3) I dont have very much work experience, but I *HAVE BEEN* in charge of a small town.4) I *GOT* some useful work experience last year.5) I *STUDIED* hard this year, so I hope I can pass my exams.6) I *HADNT BEEN* able to express myself clearly as a child. 7) I *HAVE WORKED* for more than three organizations. 8) I *KNEW* what career I wanted when I was a child. need help on this one 8. What is the sum of (5 - 7i) and (-3 + 4i)? Define Siddhartha Gautama A. found of Buddhism B. wisdom C. the Buddha's word for release from selfishness and pain D. Nirvana Find the area of a circle with a diameter of 4.Either enter an exact answer in terms of it or use 3.14 for 7 and enter your answer as a decimal.units? songs for East Africa's association officials The diameter of Earths moon is on average m. Use the formula to find the approximate surface area. (Use 3.14 for the value of .) (SHOW WORK) What is the median for the set of data? 6, 7, 10, 12, 12, 13 Answer choices:6710121213 What must be the price of a $5,000 bond with a 6.6% coupon rate, semiannual coupons, and two years to maturity if it has a yield to maturity of 10% APR? Anjali wanted to eat chick pea (Kontaikkatalai). She requested her mother to cook the same on next day. At night her mother took a cup of chick pea and put them in a container having some water and the kept the container covered overnight. Next day it was observed that the chick pea got swollen and were ready to be cooked. What is this phenomenon due to which chick pea got swollen is known as Which is most likely a source of air pollution? Phillip deposited $5,670 into a savings account 19 years ago. The account has an interest rate of 4.1% and the balance is currently $12,356.04. How often does theinterest compound? (7)* 8. Evaluate Mahesh rides a bus to work. The number of minutes his dailybus ride took over a two-week period are recorded below:M T W Th FWeek 1 12 8 10 9 12Week 2 8 7 12 9 10a. Find the mean, median, mode, and range of the data.b. Which measure would Mahesh use to describe how many minutesthe duration of his bus ride may differ from day to day? In regard to trade, the United States Terri graphed a system of linear inequalities. Which ordered pairs are a part of the solution set for this system of linear inequalities? Select two that apply.(-1, 5)(2, -4)(7, -1)(4, 6)(5, 2) Assignment: In at least two paragraphs, explain how the burst of new inventions during this erafueled the process of urbanization?